Location via proxy:   [ UP ]  
[Report a bug]   [Manage cookies]                

602 603

Download as pdf or txt
Download as pdf or txt
You are on page 1of 5

MATH 3191 Assignment 12 Solutions

Timothy Vis May 4, 2010

6.2
4 5 4 1 (6) Determine whether the set of vectors 0 , 3 8 3 3 3 , 5 is orthogonal. 1

Simply compute the inner products of all pairs of vectors in the set: (5) (4) + (4) (1) + (0) (3) + (3) (8) = 0 (5) (3) + (4) (3) + (0) (5) + (3) (1) = 0 (4) (3) + (1) (3) + (3) (5) + (8) (1) = 32. Since the last two vectors are not orthogonal, the set is not an orthogonal set. (10) Show that {u1 , u2 , u3 } is an orthogonal basis for R3 and express x as a linear combination of the us. 2 1 5 3 u1 = 3 , u2 = 2 , u3 = 1 , x = 3 1 4 1 0 To show that {u1 , u2 , u3 } is an orthogonal basis for R3 , we compute the inner products of each pair of vectors. Since all three vectors are nonzero, if they form an orthogonal set, they must be linearly independent and therefore a basis for R3 . u1 u2 = (3) (2) + (3) (2) + (0) (1) = 0 u1 u3 = (3) (1) + (3) (1) + (0) (4) = 0 u2 u3 = (2) (1) + (2) (1) + (1) (4) = 0 Since all three vectors are nonzero, and they form an orthogonal set, they must be linearly independent and therefore a basis for R3 since there are three vectors. To compute the weights in the linear combination we simply compute c1 =
xui ui ui

for each i.

(5) (3) + (3) (3) + (1) (0) x u1 4 = = 2 2 2 u1 u1 3 (3) + (3) + (0) x u2 (5) (2) + (3) (2) + (1) (1) 1 c2 = = = 2 2 2 u2 u2 3 (2) + (2) + (1) c3 = x u3 (5) (1) + (3) (1) + (1) (4) 1 = = 2 2 2 u3 u3 3 (1) + (1) + (4)

Thus, x = 4 u1 + 1 u2 + 1 u3 . 3 3 3 (16) Let y = 3 9 and u = 1 2 . Compute the distance from y to the line through u and the origin.

Compute the orthogonal projection of y onto u and the component of y orthogonal to u. y= (3) (1) + (9) (2) yu 3 u = 3u = u= 2 2 6 uu (1) + (2) 3 3 6 .y y = = 9 6 3 . Compute the norm of y y . yy = Thus, the distance is 3 5. 2 3 (20) Determine whether the set 1 , 3 vectors to produce an orthonormal set. First compute the inner product of the two vectors to determine whether they are orthogonal. 2 3 1 3 + 1 3 2 3 + 2 3 (0) = 0.
2 3

2 2 (6) + (3) = 3 5.

1 3 2 3

is orthonormal. If it is only orthogonal, normalize the

Since the inner product is zero this is certainly an orthogonal set. We also compute the norm of each vector to see if the set is orthonormal. 2 3 2 2 2 1 2 2 1 = + + = 1. 3 3 3 3 2 3 1 2 2 3 1 5 2 2 2 = . + + (0) = 3 3 3 3 0 The second vector does not have norm one so the set is not orthonormal. The rst vector is already normalized, so we simply normalize the secondvector using the norm which we already 2 5 3 5 calculated to obtain the orthonormal set 1 , 2 5 . 3 5 2 0 3 (26) Suppose W is a subspace of Rn spanned by n nonzero orthogonal vectors. Explain why W = Rn . The n nonzero orthogonal vectors must form a linearly independent set. Since the set also spans W , it forms a basis of W having n vectors. Thus, the dimension of W is n, and the only ndimensional subspace of Rn is Rn itself, so that W = Rn .

6.3
(2) Given that {u1 , u2 , u3 , u4 } is an orthogonal basis for R4 , write v as the sum of two vectors, one in Span {u1 } and the other in Span {u2 , u3 , u4 }. 4 1 1 2 1 5 1 1 1 2 u1 = 1 , u2 = 1 , u3 = 2 , u4 = 1 , v = 3 . 3 2 1 1 1 Since {u1 , u2 , u3 , u4 } is an orthogonal basis for R4 , this simply amounts to nding the orthogonal projection of v onto u1 , as Span {u2 , u3 , u4 } = (Span {u1 }) . Thus, we decompose v = v + z, with v in Span {u1 } and z in Span {u2 , u3 , u4 }. 2 4 v u1 (4) (1) + (5) (2) + (3) (1) + (3) (1) v= u1 = 2u1 = . u1 = 2 2 2 2 2 u1 u1 (1) + (2) + (1) + (1) 2 2 1 . z =vv = 5 1 (6) Verify that {u1 , u2 } is an orthogonal set and nd the orthogonal projection of y onto Span {u1 , u2 }. 4 0 6 y = 4 , u1 = 4 , u2 = 1 . 1 1 1 To verify that {u1 , u2 } is an orthogonal set, compute the inner product. u1 u2 = (4) (0) + (1) (1) + (1) (1) = 0. Then compute the orthogonal projection y= = y u1 y u2 u1 + u2 u1 u 1 u 2 u2 (6) (4) + (4) (1) + (1) (1)
2 2 2

(4) + (1) + (1) 3 5 = u 1 + u2 2 2 6 = 4 . 1

u1 +

(6) (0) + (4) (1) + (1) (1) (0) + (1) + (1)


2 2 2

u2

(14) Find the best approximation to z by vectors of the form c1 v1 + c2 v2 . 2 2 5 4 0 2 z= 0 , v1 = 1 , v2 = 4 1 3 2

We know that the best approximation is simply the orthogonal projection of z onto Span {v1 , v2 }. We rst verify that v1 and v2 are orthogonal by computing the inner product: v1 v2 = (2) (5) +

(0) (2) + (1) (4) + (3) (2) = 0. Since v1 and v2 are orthogonal, we compute the orthogonal projection in the usual fashion. z= = = z v2 z v1 v1 + vecv2 v1 v1 v2 cdotv2 (2) (2) + (4) (0) + (0) (1) + (1) (3) (2) + (0) + (1) + (3) 1 v1 2
2 2 2 2

v1 +

(2) (5) + (4) (2) + (0) (4) + (1) (2) (5) + (2) + (4) + (2)
2 2 2 2

v2

1 0 = 1 . 2 3 2 1 5 0 (20) Let u1 = 1 , u2 = 1 , and u4 = 1 . It can be shown that u4 is not in the subspace 2 2 0 W spanned by u1 and u2 . Use this fact to construct a nonzero vector v in R3 that is orthogonal to u1 and u2 . Since u4 is not in W , the component of u4 orthogonal to W is nonzero, and we compute this from the projection u4 . We rst verify that u1 and u2 are orthogonal by computing their inner product. u1 u2 = (1) (5) + (1) (1) + (2) (2) = 0. Then, u4 = u4 u2 u4 u1 u1 + u2 u1 u1 u2 u2 (1) + (1) + (2) 1 1 = u1 u 2 6 30 1 1 u4 u4 = u4 u1 + u2 6 30 0 4 = 5
2 5

(0) (1) + (1) (1) + (0) (2)


2 2 2

u1 +

(0) (5) + (1) (1) + (0) (2) (5) + (1) + (2)


2 2 2

u2

0 Thus, any multiple of this vector will work, for instance 2 . 1 (24) Let W be a subspace of Rn with an orthogonal basis {w1 , . . . , wp }, and let {v1 , . . . , vq } be an orthogonal basis for W . (a) Explain why {w1 , . . . , wp , v1 , . . . , vq } is an orthogonal set. Since {w1 , . . . , wp } is an orthogonal basis for W , every pair of vectors wi and wj is orthogonal. Since {v1 , . . . , vq } is an orthogonal basis for W , every pair of vectors vi and vj is orthogonal. Finally, since each wi is in W and each vj is in W , every pair of vectors wi and vj is orthogonal. Thus, {w1 , . . . , wp , v1 , . . . , vq } is an orthogonal set. (b) Explain why {w1 , . . . , wp , v1 , . . . , vq } spans Rn .

Since every vector in Rn can be decomposed into the sum of a vector in W and a vector in W , every vector in Rn can be written as a linear combination of the vectors in any basis for W and any basis for W . Since {w1 , . . . , wp , v1 , . . . , vq } contains bases of W and W , it spans Rn . Show that dimW + dimW = n. Since {w1 , . . . , wp , v1 , . . . , vq } is an orthogonal set of nonzero vectors, it is linearly independent. Since it spans Rn , it must be a basis of Rn and must contain exactly n vectors. On the other hand, since this set is composed of the vectors from a basis of W and a basis of W , it must have dimW + dimW vectors. Thus, dimW + dimW = n.

You might also like